Download as pdf or txt
Download as pdf or txt
You are on page 1of 35

PROPERTY | MBL | CASE DIGESTS 2020 – WEEK 2-1

II. OWNERSHIP
A. OWNERSHIP IN GENERAL
a. DEFINITION (ART. 427)
b. KINDS OF OWNERSHIP
c. RIGHTS OF AN OWNER (ARTS. 428-430, 437)
i. USE, POSSESSION, FRUITS AND DISPOSITION
PEREZ V. EVITE (G.R. NO. L-16003, MARCH 29, 1961)

PETITIONER: Cesareo Perez and Mamerta RESPONDENT: Vincente Evite and Susana
Alcantara (LOST) Manigbas
- Claimed that the decision to be executed - Prayed to the court to hold petitioners in
merely declared defendants the owners of the contempt for resisting its lawful order to
property and not to order its delivery to them deliver possession of the land
- Claimed that the adjudication of ownership
does not include possession of the property

SPECIAL DETAILS:
● Parcel of land- 11 meters wide and 37 meters long, with a total area of 407 sq. meters.

LAW & PRINCIPLES:

Under Section 45 of Rule 39, Rules of Court, which reads:


"SEC. 45. What is deemed to have been adjudged. — That only is deemed to have been adjudged in a
former judgment which appears upon its face to have been so adjudged, or which was actually and
necessarily included therein or necessary thereto,"

FACTS:
● Petitioner filed an action to quiet title against respondents.The Court of First Instance of
Batangas dismissed the complaint of plaintiffs and declared the defendants owner of the land.
● CA DECISION: affirmed the decision of the court of first instance and ordered a writ of execution
commanding the Provincial Sheriff of Batangas "to deliver the ownership of the portion of the
land in litigation to the defendant Vicente Evite, of Rosario, Batangas, pursuant to the terms and
conditions contained in the above-quoted decision."
● Plaintiffs moved to quash the writ on the ground that the decision sought to be executed merely
declared defendants owners of the property, and did not order its delivery to said parties, the writ
putting them in possession thereof was at variance with the decision and consequently, null and
void.
● This motion was denied. Plaintiffs then filed an urgent ex-parte motion "for clarification and/or to
declare null and void the Sheriff's execution. After hearing the motion, during which the
complained officials testified and explained their actuations, the court declared the same with
legal effect and valid, and dismissed plaintiffs' motion.
● Defendants' pray to the court to declare plaintiffs in contempt for resisting to deliver possession
of the land, which precipitated the issuance of July 30, 1959 order allowing defendants "to
surround their property with a fence and any act or acts by other persons or parties
including the plaintiffs to intervene may be considered as an act of contempt.”
● Plaintiffs, in resisting the trial court's orders upon the theory that the adjudication of ownership
does not include possession of the property, rely upon two cases decided by this Court.

ISSUES: WoN the ownership includes possession of the property

Page 1 of 35
PROPERTY | MBL | CASE DIGESTS 2020 – WEEK 2-1

RULING:
● It is thus evident that the pronouncement was made having in mind cases wherein the actual
possessor has a valid right over the property enforceable even against the owner thereof. As
example, we gave the cases of tenants and lessees. However, it is our view that the above
doctrines may not be invoked in instances where no such right may be appreciated in
favor of the possessor. In the instant case there appears in the appealed order of June 30,
1959, the specific finding of the trial court that "the plaintiffs have not given any reason why they
are retaining the possession of the property".
● A judgment is not confined to what appears upon the face of the decision, but also those
necessarily included therein or necessary thereto. Thus, in a land registration case wherein
ownership was adjudged, we allowed the issuance of a writ of demolition (to remove the
improvement existing on the land), for being necessarily included in the judgment.
● Considering that herein petitioners have no other claim to possession of the property apart from
their claim of ownership which was rejected by the lower court and, consequently, has no right
to remain thereon after such ownership was adjudged to defendants-appellees, the
delivery of possession of the land should be considered included in the decision.
● Indeed, it would be defeating the ends of justice should we require that for herein appellees to
obtain possession of the property duly adjudged to be theirs, from those who have no right to
remain therein, they must submit to court litigation anew.

NOTES:

Talens vs. Garcia quoting Section 45 of Rule 39, the Court said: (cited by petitioner)
"It may be admitted that the judgment absolving defendant Talens was in effect a declaration that the
sale to him was valid. It may also be admitted, though with some reluctance or reservation, that it was a
declaration of ownership of the lot. But it is doubtful whether it also included a direction to surrender it to
him. Although it is true that the owner is generally entitled to possession, it is equally true that there may
be cases where the actual possessor has some rights which must be respected or defined. A lessee is
not the owner; yet a declaration of ownership in another person does not necessarily mean his ouster."

Jabon, et al., vs. Alo, et al., the following pronouncement is found: (cited by petitioner)

" In the absence of any other declaration, can we consider a mere declaration of ownership as necessarily
including the possession of the property adjudicated? We do not believe so, for ownership is different
from possession. A person may be declared owner, but he may not be entitled to possession.
The possession may be in the hands of another either as a lessee or a tenant. A person may have
improvements thereon of which he may not be deprived without due hearing. He may have other valid
defenses to resist surrender of possession. We, therefore, hold that a judgment for ownership, does
not necessarily include possession as a necessary incident."

Mencias Case
To require a successful litigant in a land registration case to institute another action for the purpose of
obtaining possession of the land adjudged to him, would be a cumbersome process. It would foster
unnecessary and expensive litigations and result in multiplying of suits, which our judicial system abhors.

". . . Pursuant to the provision just quoted (Sec. 6, Rule 124), respondent Judge has the power to issue
all auxiliary writs, including the writ of demolition sought by petitioner, processes and other means
necessary to carry into effect the jurisdiction conferred upon it by law in land registration cases to issue
a writ of possession to the successful litigant, the petitioner herein."

Page 2 of 35
PROPERTY | MBL | CASE DIGESTS 2020 – WEEK 2-1

OLEGO VS. REBUENO (G.R. NO. L-39350, OCTOBER 29, 1975)

PETITIONER: Cenona Olego WON RESPONDENT: Hon. Alfredo Rebueno and


- Claimed to be the owner of the subject lot Atty. Pedro Servano
- denied that Servano owned and - averred that Cenona Olego's claim of
possessed any land located at Sta. Cruz ownership caused him damages in the sum
Street, Lagonoy of P300 as attorney's fees and P120
- Claimed that Servano had never annually as rentals which he was not able
possessed it nor manifested his claim of to collect because of her unjustified claim
ownership; of ownership.
- Claimed that she inherited the land from - prayed that his "title" be declared "legal,"
her late father and that her predecessors that he be adjudged as the absolute owner
were of possession of the said land since of the land, entitled to its "peaceful
time immemorial possession
- prayed that Cenona Olego and her children
be punished for contempt of court and
ordered to pay damages to him
- alleged that Cenona Olego and her two
children prevented him and his agents from
enjoying "the peaceful use and
possession”

SPECIAL DETAILS:
● Pedro D. Servano is a lawyer from Naga City.

LAW & PRINCIPLES:

The general rule is that the adjudication of ownership does not include the possession of
the property. The exception is that the adjudication of ownership would include the
delivery of possession if the defeated party has not shown any right to possess the land
independently of his claim of ownership which was rejected.

In such a case, a writ of execution would be required if the defeated party does not surrender
the possession of the property. The owner should enforce his right to possess the land (as
an incident of his ownership) by asking for a writ of execution within five years from the finality
of the decision. Thereafter, he could enforce his right by action within the next five years.

FACTS:
● Pedro D. Servano filed a complaint against Cenona Olego and asked for a declaration as to the
legality of his title to a residential lot with an area of 1,225 square meters located at Sta. Cruz
street, Lagonoy, Camarines Sur, supposedly covered by a tax declaration in his name.
● He alleged that he acquired the lot by purchase and that he and his predecessors had possessed
it en concepto de dueño since time immemorial.
● After he had acquired the lot, Cenona Olego claimed to be the owner of the lot, "thereby casting
a cloud of doubt" on his title and rendering it necessary that his "title and possession" "be
declared legal."
● He also averred that Cenona Olego's claim of ownership caused him damages in the sum of
P300 as attorney's fees and P120 annually as rentals which he was not able to collect because
of her "unjustified claim of ownership.
● Olego denied that Servano owned and possessed any land located at Sta. Cruz Street, Lagonoy.
She pleaded the defense that she was the "absolute owner and lawful possessor" of the land

Page 3 of 35
PROPERTY | MBL | CASE DIGESTS 2020 – WEEK 2-1

located at Sta. Cruz Street, Lagonoy and is covered by Tax Declaration No. 255 in the name of
Santiago Olego.
● She averred that if Servano's complaint referred to that land, then he had "absolutely no right
whatsoever over the land"; that he had never possessed it nor manifested his claim of ownership;
that she inherited the land from her late father, Santiago Olego.
● She and her predecessors had been in actual possession of the land en concepto de dueño
since time immemorial, and that Servano's claim was "unfounded and without any legal basis".
She filed a counterclaim.
● The parties signed an amicable agreement stating that Olego admits the allegation of Servano
and renders him as the owner of the land in question and that Servano waves his claim for
attorney’s fees and damages against Olego. (1964 Decision)
● The record shows that there was no further proceeding in the case more than ten years after the
alleged compromise settlement was concluded.
● After that decade of inactivity, Servano filed in the lower court a "petition for contempt" against
Cenona Olego and her children wherein he alleged that he was the prevailing party and Cenona
Olego the defeated party in the decision which allegedly was "immediately executory", Cenona
Olego and her children prevented him from having "the peaceful use and enjoyment" of said
land, that they branded the decision as "fake", and that they occupied a portion of the land and
disturbed Servano's "property rights."
● The record does not show that respondents Cenona Olego and her two children were served
copies of the petition which was dated February 19, 1974.
● Judge Rebueno issued the order that the 1964 decision is already long final and executory and
that they have no more right to remain in the property.
● Servano asked that Cenona Olego and her children, who, despite Judge Rebueno's explanation,
still persisted in not recognizing the lower court's decision and who had threatened to injure the
persons who would dispossess them of the land, be given only fifteen days to remove their "small
nipa huts.”
● Cenona Olego through a new counsel, filed a motion to dismiss the contempt charge on the
grounds (a) that the amicable settlement was obtained through fraud and misrepresentation, (b)
that the execution of the judgment was barred by the statute of limitations, and (c) that the court
had lost jurisdiction over the case.
● it was pointed out in the motion that the judgment does not require Cenona Olego to vacate the
land or deliver its possession to Servano; that, being illiterate, she was unaware that she had
signed an amicable settlement; that she was never furnished with copies of both the amicable
settlement and the 1964 decision, she was made to understand that what she was signing was
a motion for postponement, and that, being ignorant of the decision, she never vacated the land
and she thought that the case was still to be tried.
● Servano did nothing to implement the decision within the ten-year period for enforcing it; that he
never occupied any portion of the land, and that Cenona Olego came to know of the decision
when she was being charged with contempt of court. Atty. Flores contended that the lower court
had no more jurisdiction over the case in view of the expiration of the ten-year period for enforcing
its judgment.
● Atty. Servano in his comment alleged that the decision was "self-executory"; that Cenona Olego
delivered the possession of the lot to him, and that she and her children later re-entered the lot.

ISSUES:
1. whether Cenona Olego could be held in contempt of court for not vacating the land
involved in the compromise
2. whether the lower court's decision should be set aside on the ground of fraud or could
be enforced after the expiration of more than ten years from the date of its finality

RULING:
1. Cenona Olego's failure to vacate the lot could not be the basis of a contempt proceeding against
her. The orders citing her for contempt are oppressive, unjust and unwarranted. In the

Page 4 of 35
PROPERTY | MBL | CASE DIGESTS 2020 – WEEK 2-1

compromise agreement and in the decision approving it, she was not ordered to vacate the
lot. It was stipulated in the compromise that she admitted Atty. Servano's ownership of the lot
and "that judgment be rendered declaring" him to be the owner thereof. The decision approving
the compromise followed the usual pattern of judgments in such cases: ordering the parties to
comply with the terms and stipulations of the compromise.

The rule is that an order or judgment which declares the rights of the parties without any express
command or prohibition is not one which may be the basis of a contempt proceeding. A violation
of the rights of ownership does not constitute contempt of court, even though they have
been ascertained and declared by judgment, unless it consists in doing something that was
prohibited, or in falling to do something that was required, by the terms of the judgment.
Where there is no decree or order commanding accused or anyone else to do or refrain from
doing something or anything, disobedience of it is impossible. Hence, Cenona Olego could not
be held guilty of contempt of court.

2. A compromise may be annulled on the ground of fraud and mistake. A judicial compromise may
be set aside if fraud vitiated the consent of a party thereof. The extrinsic fraud, which nullifies a
compromise, likewise invalidates the decision approving it.

As to the enforceability of the lower court's decision, it should be noted that assuming arguendo
that that decision is valid, it becomes necessary to find out whether the decision includes the
delivery of the possession of the land to Atty. Servano. The decision was based on Cenona
Olego's supposed admission that Atty. Servano is the owner of the land in question.

Does Cenona Olego's acknowledgment of Servano's ownership include the obligation to deliver
the possession of the land to him? The general rule is that the adjudication of ownership
does not include the possession of the property. The exception is that the adjudication of
ownership would include the delivery of possession if the defeated party has not shown
any right to possess the land independently of his claim of ownership which was rejected.

In such a case, a writ of execution would be required if the defeated party does not surrender
the possession of the property. The owner should enforce his right to possess the land (as
an incident of his ownership) by asking for a writ of execution within five years from the finality
of the decision. Thereafter, he could enforce his right by action within the next five years.

Atty. Servano did not enforce his right to possess the land within the ten-year period. To
enforce the judgment in his favor by means of a contempt proceeding after the expiration of the
ten-year period would be a circumvention of the statute of limitations. What the law prohibits
directly should not be allowed to be done indirectly.

The judgement against Cenona Olego is no longer enforceable and cannot possibly affect her
possession on the disputed land.

NOTES:

The judgment against Cenona Olego, which had become unenforceable by reason of prescription, is a
dormant judgment. The dormancy of a judgment destroys its legal force and effect.

NAZARENO VS. COURT OF APPEALS (G.R. NO. 138842, OCTOBER 18, 2000)

Page 5 of 35
PROPERTY | MBL | CASE DIGESTS 2020 – WEEK 2-1

PETITIONER: Natividad Nazareno, Maximo RESPONDENT: CA, Estate of Maximino


Nazareno Jr. Nazareno Sr., Romeo Nazareno, Eliza Nazareno
- Sought the declaration nullity of the sale - They alleged that Lot 3 is being leased by
made to Natividad and that made to the spouses Romeo and Eliza to third
Maximino, Jr. on the ground that both persons.
sales were void for lack of consideration. - They sought the annulment of the transfer
to Romeo and the cancellation of his title,
the eviction of Romeo and his wife Eliza
and all persons claiming rights from Lot 3,
and the payment of damages.

SPECIAL DETAILS:
● Maximino Nazareno, Sr. and Aurea Poblete were husband and wife. Aurea died on April 15,
1970, while Maximino, Sr. died on December 18, 1980. They had five children, namely,
Natividad, Romeo, Jose, Pacifico, and Maximino, Jr. Natividad and Maximino, Jr. are the
petitioners in this case, while the estate of Maximino, Sr., Romeo, and his wife Eliza Nazareno
are the respondents.
● Romeo received the title to Lot 25-L under his name, while Maximino, Jr. received Lots 6 and 7
through a Deed of Sale for the amount of P9,500.00. Pacifico and Jose's shares were allegedly
given to Natividad, who agreed to give Lots 10 and 11 to Jose, in the event the latter came back
from abroad.

LAW & PRINCIPLES:


Suntay vs. Court of Appeals
The Supreme Court held that badges of simulation make a deed of sale null and void since parties thereto
enter into a transaction to which they did not intend to be legally bound.

FACTS:
● Maximino Nazareno, Sr. and Aurea Poblete acquired properties in Quezon City and in the
Province of Cavite. It is the ownership of some of these properties that is in question in this case.
● Romeo filed an intestate case in the Court of First Instance of Cavite. Upon the reorganization
of the courts in 1983, the case was transferred to the Regional Trial Court of Naic, Cavite. Romeo
was appointed administrator of his father's estate.
● In the course of the intestate proceedings, Romeo discovered that his parents had executed
several deeds of sale conveying a number of real properties in favor of his sister, Natividad. One
of the deeds involved six lots in Quezon City which were allegedly sold by Maximino, Sr., with
the consent of Aurea, to Natividad for the total amount of P47,800.00.
● Among the lots covered by the Deed of Sale is Lot 3-B. This lot had been occupied by Romeo,
his wife Eliza, and by Maximino, Jr. since 1969. Unknown to Romeo, Natividad sold Lot 3-B on
July 31, 1982 to Maximino, Jr., for which a TCT was issued by the Register of Deeds of Quezon
City.
● When Romeo found out about the sale to Maximino, Jr., he and his wife Eliza locked Maximino,
Jr. out of the house. Maximino, Jr. brought an action for recovery of possession and damages
with prayer for writs of preliminary injunction and mandatory injunction. The trial court ruled in
favor of Maximino, Jr. The Court of Appeals affirmed the decision of the trial court.
● Romeo in turn filed, on behalf of the estate of Maximino, Sr., the present case for annulment of
sale with damages against Natividad and Maximino, Jr. The case was filed in the Regional Trial
Court of Quezon City. Romeo sought the declaration of nullity of the sale made to Natividad and
that made to Maximino, Jr. on the ground that both sales were void for lack of consideration.
● Natividad and Maximino, Jr. filed a third-party complaint and alleged that Lot 3, which was
included in the Deed of Absolute Sale to Natividad, had been surreptitiously appropriated by
Romeo by securing for himself a new title in his name. They alleged that Lot 3 is being leased
by the spouses Romeo and Eliza to third persons. They sought the annulment of the transfer to

Page 6 of 35
PROPERTY | MBL | CASE DIGESTS 2020 – WEEK 2-1

Romeo and the cancellation of his title, the eviction of Romeo and his wife Eliza and all persons
claiming rights from Lot 3, and the payment of damages.
● Romeo presented evidence to show that Maximino and Aurea Nazareno never intended to sell
the six lots to Natividad and that Natividad was only to hold the said lots in trust for her siblings.
He presented the Deed of Partition and Distribution.
● Romeo testified that, although the deeds of sale executed by his parents in their favor stated that
the sale was for a consideration, they never really paid any amount for the supposed sale. The
transfer was made in this manner in order to avoid the payment of inheritance taxes. Romeo
denied stealing Lot 3 from his sister but instead claimed that the title to said lot was given to him
by Natividad in 1981 after their father died.
● Respondents claimed that the Deed of Partition and Distribution executed in 1962 was not
carried out. Instead, their parents offered to sell to them the six lots in Quezon City which
Natividad bought because she was the only one financially able to do so. Natividad admitted that
Romeo and the latter's wife were occupying Lot 3-B at that time and that she did not tell the latter
about the sale she had made to Maximino, Jr.
● Natividad said that she had the title to Lot 3 but it got lost. She could not get an original copy of
the said title because the records of the Registrar of Deeds had been destroyed by fire. She
claimed she was surprised to learn that Romeo was able to obtain a title to Lot 3 in his name.
● CA modified the decision of the trial court that titles to Lot 3 and Lot 3-B , as well as to Lots 10
and 11 were cancelled and ordered restored to the estate of Maximino Nazareno, Sr.

ISSUES: Whether or not the Deed of Sale is valid thus vesting Natividad ownership (NO)

RULING:
● The fact that the deed of sale was notarized is not a guarantee of the validity of its contents.
● Petitioners allege that, as shown by several deeds of sale executed by Maximino, Sr. and Aurea
during their lifetime, the intention to dispose of their real properties is clear. Consequently, they
argue that the Deed of Sale should also be deemed valid.This is a non-sequitur. The fact that
other properties had allegedly been sold by the spouses Maximino, Sr. and Aurea does
not necessarily show that the Deed of Sale made on January 29, 1970 is valid.
● The trial court and the Court of Appeals found that the Nazareno spouses transferred their
properties to their children by fictitious sales in order to avoid the payment of inheritance taxes.
Indeed, it was found both by the trial court and by the Court of Appeals that Natividad had no
means to pay for the six lots subject of the Deed of Sale.
● Facts and circumstances indicate badges of a simulated sale which make the Deed of Absolute
Sale void and of no effect. In the case of Suntay vs. Court of Appeals, the Supreme Court held
that badges of simulation make a deed of sale null and void since parties thereto enter
into a transaction to which they did not intend to be legally bound.
● It appears that it was the practice in the Nazareno family to make simulated transfers of
ownership of real properties to their children in order to avoid the payment of inheritance taxes.
Per the testimony of Romeo, he acquired Lot 25-L from his parents through a fictitious or
simulated sale wherein no consideration was paid by him. He even truthfully admitted that the
sale of Lot 3 to him on 04 July 1969 likewise had no consideration. This document was signed
by the spouses Max, Sr. and Aurea as vendors while defendant-appellant Natividad signed as
witness.
● As Romeo admitted, no consideration was paid by him to his parents for the Deed of Sale.
Therefore, the sale was void for having been simulated. Natividad never acquired ownership
over the property because the Deed of Sale in her favor is also void for being without
consideration and title to Lot 3 cannot be issued in her name.
● Nonetheless, it cannot be denied that Maximino, Sr. intended to give the six Quezon City lots to
Natividad. Their parents executed the Deed of Sale in favor of Natividad because the latter was
the only "female and the only unmarried member of the family." She was thus entrusted with the
real properties in behalf of her siblings. As she herself admitted, she intended to convey Lots 10

Page 7 of 35
PROPERTY | MBL | CASE DIGESTS 2020 – WEEK 2-1

and 11 to Jose in the event the latter returned from abroad. There was thus an implied trust
constituted in her favor.

NOTES:

Suntay v. Court of Appeals:


Though the notarization of the deed of sale in question vests in its favor the presumption of regularity, it
is not the intention nor the function of the notary public to validate and make binding an instrument never,
in the first place, intended to have any binding legal effect upon the parties thereto. The intention of the
parties still and always is the primary consideration in determining the true nature of a contract.

FLANCIA VS. COURT OF APPEALS (G.R. NO. 146997, APRIL 26, 2005)

PETITIONER: Sps. Godofredo and Dominica Flancia RESPONDENT: CA and


- alleged that the mortgage of subject lot is null and void as it is William Ong Genato (WON)
not authorized by plaintiffs pursuant to Art. 2085 of the Civil - plaintiffs have no
Code which requires that the mortgagor must be the absolute cause of action against
owner of the mortgaged property. defendant Genato.
- advised defendants to exclude subject lot from the auction sale

SPECIAL DETAILS:
● A parcel of land known as Lot 12, Blk. 3, Phase III-A containing an area of 128.75 square meters
situated in Prater Village Subd. II located at Brgy. Old Balara, Quezon City.
● What was stated in the contract between Oakland and petitioners:
- BUYER/S may be allowed to enter into and take possession of the property upon
issuance of Occupancy Permit by the OWNER/DEVELOPER exclusively, although title
has not yet passed to the BUYER/S
- the BUYER/S cannot sell, mortgage, cede, transfer, assign or in any manner
alienate or dispose of
- That this Contract to Sell shall not in any way authorize the BUYER/S to occupy the
assigned house and lot to them.

LAW & PRINCIPLES:


Under the Art. 2085 of the Civil Code, the essential requisites of a contract of mortgage are: (a) that it
be constituted to secure the fulfillment of a principal obligation; (b) that the mortgagor be the absolute
owner of the thing mortgaged; and (c) that the persons constituting the mortgage have the free disposal
of their property, and in the absence thereof, that they be legally authorized for the purpose.

Difference between a Contract of Sale and a Contract to Sell


In a contract of sale, title to the property passes to the vendee upon the delivery of the thing sold; in a
contract to sell, ownership is, by agreement, reserved by the vendor and is not to pass to the vendee
until full payment of the purchase price.

Otherwise stated, in a contract of sale, the vendor loses ownership over the property and cannot recover
it unless and until the contract is resolved or rescinded; in a contract to sell, title is retained by the vendor
until full payment of the price.

FACTS:

Page 8 of 35
PROPERTY | MBL | CASE DIGESTS 2020 – WEEK 2-1

● Plaintiffs allege that they purchased from defendant corporation a parcel of land located at Brgy.
Old Balara, Quezon City; that by virtue of the contract of sale, defendant corporation authorized
plaintiffs to transport all their personal belongings to their house at the aforesaid lot.
● On December 24, 1992, plaintiffs received a copy of the execution foreclosing the mortgage
issued by the RTC ordering defendant Sheriff Sula to sell at public auction several lots formerly
owned by defendant corporation including subject lot of plaintiffs.
● They alleged that the mortgage of subject lot is null and void as it is not authorized by plaintiffs
pursuant to Art. 2085 of the Civil Code which requires that the mortgagor must be the absolute
owner of the mortgaged property.
● As a consequence of the nullity of said mortgage, the execution foreclosing mortgage is likewise
null and void; that plaintiffs advised defendants to exclude subject lot from the auction sale but
the latter refused. Plaintiffs likewise prayed for damages in the sum of P50,000.00.
● Defendant William Ong Genato filed a motion to dismiss the complaint. He filed his answer
averring that co-defendant Oakland Development Resources Corporation mortgaged to Genato
two (2) parcels of land as security and guaranty for the payment of a loan in the sum of
P2,000,000.00.
● The said real estate mortgage has been duly annotated and that for the non-payment of the loan
of P2,000,000.00. Genato filed an action for foreclosure of real estate mortgage against co-
defendant corporation.
● The RTC rendered a decision against defendant corporation and was affirmed by the CA. RTC
issued an order to cause the sale at public auction of the said properties for failure of defendant
corporation to deposit in Court the money judgement.
● The plaintiffs have no cause of action against defendant Genato. The alleged plaintiffs' Contract
to Sell does not appear to have been registered with the Register of Deeds of Quezon City to
affect defendant Genato and the latter is thus not bound by the plaintiffs' Contract to Sell.
● The registered mortgage is superior to plaintiffs' alleged Contract to Sell and it is sufficient for
defendant Genato as mortgagee to know that the TCT was clean at the time of the execution of
the mortgage contract with defendant corporation and defendant Genato is not bound to go
beyond the title to look for flaws in the mortgagor's title.
● Plaintiffs' alleged Contract to Sell is neither a mutual promise to buy and sell nor a Contract of
Sale. Ownership is retained by the seller, regardless of delivery and is not to pass until full
payment of the price.
● Genato did not receive any advice from plaintiffs to exclude the subject lot from the auction sale.

TRIAL COURT: ordered Oakland Dev’t Resources to pay the plaintiffs.


On Motion for Reconsideration (1996): declared the subject mortgage as null and void and to pay the
plaintiffs.
CA: Reversed the 1996 Decision of the trial court.

ISSUES:
1. whether or not the registered mortgage constituted over the property was valid (YES)

2. whether or not the registered mortgage was superior to the contract to sell (YES)

3. whether or not the mortgagee was in good faith.

RULING:
● All the requisites in Art. 2085 are present in the case.

1. As to the first essential requisite of a mortgage, it is undisputed that the mortgage was executed
on May 15, 1989 as security for a loan obtained by Oakland from Genato.

In a contract of sale, title to the property passes to the vendee upon the delivery of the thing sold;
in a contract to sell, ownership is, by agreement, reserved by the vendor and is not to pass to

Page 9 of 35
PROPERTY | MBL | CASE DIGESTS 2020 – WEEK 2-1

the vendee until full payment of the purchase price.Otherwise stated, in a contract of sale, the
vendor loses ownership over the property and cannot recover it unless and until the contract is
resolved or rescinded; in a contract to sell, title is retained by the vendor until full payment of the
price.

In the contract between petitioners and Oakland, aside from the fact that it was denominated as
a contract to sell, the intention of Oakland not to transfer ownership to petitioners until full
payment of the purchase price was very clear. Acts of ownership over the property were
expressly withheld by Oakland from petitioner. All that was granted to them by the "occupancy
permit" was the right to possess it.

Aside from the jus utendi and the jus abutendi inherent in the right to enjoy the thing, the right to
dispose, or the jus disponendi, is the power of the owner to alienate, encumber, transform and
even destroy the thing owned. Because Oakland retained all the foregoing rights as owner of the
property, it was entitled absolutely to mortgage it to Genato. Hence, the mortgage was valid.

2. State Investment House is completely inapplicable to the case at bar. A contract of sale and a
contract to sell are worlds apart. State Investment House clearly pertained to a contract of sale,
not to a contract to sell which was what Oakland and petitioners had. In State Investment House,
ownership had passed completely to the buyers and therefore, the former owner no longer had
any legal right to mortgage the property, notwithstanding the fact that the new owner-buyers had
not registered the sale. In the case before us, Oakland retained absolute ownership over the
property under the contract to sell and therefore had every right to mortgage it.

In sum, we rule that Genato's registered mortgage was superior to petitioner's contract to
sell, subject to any liabilities Oakland may have incurred in favor of petitioners by irresponsibly
mortgaging the property to Genato despite its commitments to petitioners under their contract to
sell.

3. This Court is not a trier of facts. The resolution of factual issues is the function of the lower courts.
We therefore adopt the factual findings of the Court of Appeals and uphold the good faith of the
mortgagee Genato.

● Just as an innocent purchaser for value may rightfully rely on what appears in the certificate of
title, a mortgagee has the right to rely on what appears in the title presented to him. In the
absence of anything to arouse suspicion, he is under no obligation to look beyond the certificate
and investigate the title of the mortgagor appearing on the face of the said certificate.

NOTES:

VDA. DE BAUTISTA VS. MARCOS (G.R. NO. L-17072, OCTOBER 31, 1961)

PETITIONER: Cristina Marcelo Vda. De RESPONDENT: Brigida Marcos


Bautista - Claimed that the land in question is covered by a
- filed the present action against Free Patent and cannot be taken within 5 years
Brigida and her husband for the from the issuance of the patent for the payment
payment thereof, or in default of the of any debts of the patentees contracted prior to
debtors to pay, for the foreclosure of the expiration of said five-year period under the
her mortgage on the land given as Public Land Law.
security.

Page 10 of 35
PROPERTY | MBL | CASE DIGESTS 2020 – WEEK 2-1

SPECIAL DETAILS:
● The unregistered parcel of land is situated in Sta. Ignacia, Tarlac.
● Free Patent No. V-64358 was issued to the applicants (respondents).

LAW & PRINCIPLES:


Art. 1434, N.C.C. provides that "When a person who is not the owner of a thing sells or alienates and
delivers it, and later the seller or grantor acquires title thereto, such title passes by operation of law to
the buyer or grantee."

FACTS:
● Marcos obtained a loan in the amount of P2,000 from plaintiff Cristina Marcelo Vda. de Bautista
and to secure payment thereof conveyed to the latter by way of mortgage a two (2) hectare
portion of an unregistered parcel of land.
● The deed of mortgage, provided that it was to last for three years, that possession of the land
mortgaged was to be turned over to the mortgagee by way of usufruct, but with no obligation on
her part to apply the harvests to the principal obligation and that said mortgage would be released
only upon payment of the principal loan of P2,000 without any interest and that the mortgagor
promised to defend and warrant the mortgagee's rights over the land mortgaged.
● Mortgagor Brigida Marcos filed, on behalf of the heirs of her deceased mother Victoriana
Cainglet, an application for the issuance of a free patent over the land in question, on the strength
of the cultivation and occupation of said land by them and their predecessor since July, 1915.
● A Free Patent was issued to the applicants. It was registered in their names under OCT No. P-
888 of the office of the Register of Deeds for the province of Tarlac.
● Defendant Brigida Marcos' indebtedness to plaintiff having remained unpaid, the latter filed the
present action against Brigida and her husband for the payment thereof, or in default of the
debtors to pay, for the foreclosure of her mortgage on the land given as security.
● Defendants moved to dismiss the action, since the land in question is covered by a free patent
and could not, under the Public Land Law, be taken within five years from the issuance of the
patent for the payment of any debts of the patentees contracted prior to the expiration of said
five-year period.
● The lower court denied the motion to dismiss on the ground that the law cited does not apply
because the mortgage sought to be foreclosed was executed before the patent was issued.
● Defendants reiterating the defense invoked in their motion to dismiss, and alleging as well that
the real contract between the parties was an antichresis and not a mortgage.

LOWER COURT RULING


Rendered judgment finding the mortgage valid to the extent of the mortgagor's pro-indiviso share of
15,333 square meters in the land in question, on the theory that the Public Land Law does not apply in
this case because the mortgage in question was executed before patent was issued over the land in
question.
The agreement of the parties could not be an antichresis because the deed shows a mortgage with
usufruct in favor of the mortgagee; and ordered the payment of the mortgage loan of P2,000 to plaintiff
or, upon defendant's failure to do so, the foreclosure of plaintiff's mortgage on defendant Brigida Marcos'
undivided share in the land in question.

ISSUES: W/N the mortgage is valid and enforceable

RULING:
● As it is an essential requisite for the validity of a mortgage that the mortgagor be the absolute
owner of the thing mortgaged (Art. 2085), the mortgage here in question is void and
ineffective because at the time it was constituted, the mortgagor was not yet the owner of the
land mortgaged and could not, for that reason, encumber the same to plaintiff-appellee.

Page 11 of 35
PROPERTY | MBL | CASE DIGESTS 2020 – WEEK 2-1

● Nor could the subsequent acquisition by the mortgagor of title over said land through the
issuance of a free patent validate and legalize the deed of mortgage under the doctrine of
estoppel (cf. Art. 1434, New Civil Code, 1 ), since upon the issuance of said patent, the land in
question was thereby brought under the operation of the Public Land Law that prohibits the taking
of said land for the satisfaction of debts contracted prior to the expiration of five years from the
date of the issuance of the patent (sec. 118, C.A. No. 141).
● This prohibition should include not only debts contracted during the five-year period immediately
following the issuance of the patent but also those contracted before such issuance, if the
purpose and policy of the law, which is "to preserve and keep in the family of the homesteader
that portion of public land which the State has gratuitously given to him" is to be upheld.
● The invalidity of the mortgage does not, imply the concomitant invalidity of the collateral
agreement in the same deed of mortgage whereby possession of the land mortgaged was
transferred to plaintiff-appellee in usufruct, without any obligation on her part to account
for its harvests or deduct them from defendants' indebtedness of P2,000.
● Defendant Brigida Marcos, who, together with her sisters, was in possession of said land by
herself and through her deceased mother, had possessory rights over the same even before
title vested in her as co-owner by the issuance of the free patent to her and her sisters,
and these possessory rights, she could validly transfer and convey to plaintiff- appellee,
as she did in the deed of mortgage Exhibit "A".
● The latter, upon the other hand, believing her mortgagor to be the owner of the land mortgaged
and not being aware of any flaw which invalidated her mode of acquisition, was a possessor in
good faith (Art. 526, N.C.C.), and as such had the right to all the fruits received during the
entire period of her possession in good faith (Art. 544, N.C.C.). She is, therefore, entitled
to the full payment of her credit of P2,000 from defendants, without any obligation to
account for the fruits or benefits obtained by her from the land in question.

NOTES:

ii. DOCTRINE OF SELF-HELP


GERMAN MANAGEMENT & SERVICES VS. COURT OF APPEALS (G.R. NO. 76216. SEPTEMBER
14, 1989)

PETITIONER: RESPONDENT:

GERMAN MANAGEMENT & SERVICES Mountainside farmers occupying the disputed land
(Lose) (Win)
Property developer authorized with special
power of attorney by the lot owners to
develop their parcel of land into a
subdivision. ● Filed an action for forcible entry against
petitioner before the Municipal Trial Court of
● Petitioner advised the occupants to Antipolo, Rizal, alleging that they are
vacate the premises but the latter mountainside farmers of Sitio Inarawan, San
refused. Nevertheless, petitioner Isidro, Antipolo, Rizal and members of the
proceeded with the development of Concerned Citizens of Farmer's Association;
the subject property which included that they have occupied and tilled their
the portions occupied and cultivated farmholdings some twelve to fifteen years prior
by private respondents. to the promulgation of P.D. No. 27
● Petitioner deprived private respondents of
their property without due process of law by:
(1) forcibly removing and destroying the

Page 12 of 35
PROPERTY | MBL | CASE DIGESTS 2020 – WEEK 2-1

barbed wire fence enclosing their


farmholdings without notice; (2) bulldozing the
rice, corn fruit bearing trees and other crops of
private respondents by means of force,
violence and intimidation, in violation of P.D.
1038 and (3) trespassing, coercing and
threatening to harass, remove and eject
private respondents from their respective
farmholdings in violation of P.D. Nos. 316,
583, 815, and 1028.

SPECIAL DETAILS:
Spouses are residents of Pennsylvania, Philadelphia, USA. They are the owners of a parcel of land
situated in Sitio Inarawan, San Isidro, Antipolo, Rizal, with an area of 232,942 square meters

LAW & PRINCIPLES: Art. 429 of the Civil Code; does not apply in the case at bar; When possession
has already been lost, the owner must resort to judicial process for the recovery of property.

FACTS:
Spouses Jose are the owners of a parcel of land in Antipolo. They executed a special power of attorney
authorizing German management Services to develop their property into a residential subdivision.
However, the property was being occupied by private respondents and twenty other persons. They
were asked to vacate but refused.

Private respondents filed an action for forcible entry and alleged that they are mountainside farmers of
the area and have occupied and tilled their farmholdings prior to the promulgation of PD 27. They
stated that they have been deprived of their property without due process of law by means of force,
violence and intimidation.

ISSUES: Whether or not private respondents are entitled to file a forcible entry case against
petitioner?

RULING: YES.

Since private respondents were in actual possession of the property at the time they were forcibly ejected
by petitioner, private respondents have a right to commence an action for forcible entry regardless of the
legality or illegality of possession.

Private respondents, as actual possessors, can commence a forcible entry case against petitioner
because ownership is not in issue. Forcible entry is merely a quieting process and never determines the
actual title to an estate. Title is not involved, only actual possession. It is undisputed that private
respondents were in possession of the property and not the petitioners nor the spouses Jose. Although
the petitioners have a valid claim over ownership this does not in any way justify their act of forcible entry.

NOTES:

Both the Municipal Trial Court and the Regional Trial Court have rationalized petitioner's drastic action
of bulldozing and destroying the crops of private respondents on the basis of the doctrine of self-help
enunciated in Article 429 of the New Civil Code.

Such justification is unavailing because the doctrine of self-help can only be exercised at the
time of actual or threatened dispossession which is absent in the case at bar. When possession has
already been lost, the owner must resort to judicial process for the recovery of property. This is clear

Page 13 of 35
PROPERTY | MBL | CASE DIGESTS 2020 – WEEK 2-1

from Article 536 of the Civil Code which states, "(I)n no case may possession be acquired through
force or intimidation as long as there is a possessor who objects thereto. He who believes that he
has an action or right to deprive another of the holding of a thing, must invoke the aid of the
competent court, if the holder should refuse to deliver the thing."

iii. ENCLOSING AND FENCING (ART. 430)


iv. SURFACE RIGHTS OF LANDOWNER (ART.
437)
1. EXTENT
2. LIMITATIONS
3. CASE

REPUBLIC VS. COURT OF APPEALS (G.R. NO. L-43938. APRIL 15, 1988)

PETITIONER: RESPONDENT:

Applicants Jose de la Rosa on his own behalf and Court of Appeals, who erroneously applied the
on behalf of his three children, Victoria, Benjamin Regalian doctrine.
and Eduard, who sought to register a parcel of land in
Benguet Province. Under the theory of the respondent court, the
surface owner will be planting on the land while
Benguet Consolidated Inc. and Atok-Big Wedge
Mining Company who opposed such registration on
the mining locator will be boring tunnels
the ground that they have mineral claims over the land. underneath.

SPECIAL DETAILS: This is a consolidated case wherein the Regalian doctrine was not correctly
applied.

LAW & PRINCIPLES: The owner of a piece of land has rights not only to its surface but also to everything
underneath and the airspace above it up to a reasonable height. The rights over the land are indivisible
and the land itself cannot be half agricultural and half mineral. The classification must be categorical; the
land must be either completely mineral or completely agricultural.

FACTS:

These cases arose from the application for registration of a parcel of land filed on February 11, 1965, by Jose de la
Rosa on his own behalf and on behalf of his three children, Victoria, Benjamin and Eduardo. The land, situated in
Tuding, Itogon, Benguet Province, was divided into 9 lots.

Lots 1-5 were sold to Jose de la Rosa and Lots 6-9 to his children by Mamaya Balbalio and Jaime Alberto.

The application was separately opposed by Benguet Consolidated, Inc. as to Lots 1-5, Atok Big Wedge Corporation,
as to Portions of Lots 1-5 and all of Lots 6-9, and by the Republic of the Philippines, through the Bureau of Forestry
Development, as to lots 1-9.

Benguet opposed on the ground that the June Bug mineral claim covering Lots 1-5 was sold to it. . From the date
of its purchase, Benguet had been in actual, continuous and exclusive possession of the land in concept of owner,
as evidenced by its construction of adits, its affidavits of annual assessment, its geological mappings, geological
samplings and trench side cuts, and its payment of taxes on the land.

For its part, Atok alleged that a portion of Lots 1-5 and all of Lots 6-9 were covered by the Emma and Fredia mineral
claims located by Harrison and Reynolds. These claims were purchased by Atok, which has since then been in
open, continuous and exclusive possession of the said lots as evidenced by its annual assessment work on the
claims, such as the boring of tunnels, and its payment of annual taxes thereon.

Page 14 of 35
PROPERTY | MBL | CASE DIGESTS 2020 – WEEK 2-1

The Bureau of Forestry Development also interposed its objection, arguing that the land sought to be registered by
reason of its nature, it was not subject to alienation under the Constitutions of 1935 and 1973.

The trial court denied the application, holding that the applicants had failed to prove their claim of possession and
ownership of the land sought to be registered.

The applicants appealed to the respondent court, which reversed the trial court and recognized the claims of the
applicant, but subject to the rights of Benguet and Atok respecting their mining claims. In other words, the Court of
Appeals affirmed the surface rights of the de la Rosas over the land while at the same time reserving the sub-surface
rights of Benguet and Atok by virtue of their mining claims. Both Benguet and Atok have appealed to this Court,
invoking their superior right of ownership.

ISSUES: Whether respondent court’s decision, i.e. “the surface rights of the de la Rosas over the land while at
the same time reserving the sub-surface rights of Benguet and Atok by virtue of their mining claim,” is correct.

RULING: No.

Our holding is that Benguet and Atok have exclusive rights to the property in question by virtue of their
respective mining claims which they validly acquired before the Constitution of 1935 prohibited the
alienation of all lands of the public domain except agricultural lands, subject to vested rights existing at
the time of its adoption. The land was not and could not have been transferred to the private respondents
by virtue of acquisitive prescription, nor could its use be shared simultaneously by them and the mining
companies for agricultural and mineral purposes. It is true that the subject property was considered forest
land and included in the Central Cordillera Forest Reserve, but this did not impair the rights already
vested in Benguet and Atok at that time. Such rights were not affected either by the stricture in the
Commonwealth Constitution against the alienation of all lands of the public domain except those
agricultural in nature for this was made subject to existing rights. The perfection of the mining claim
converted the property to mineral land and under the laws then in force removed it from the public
domain. By such act, the locators acquired exclusive rights over the land, against even the government,
without need of any further act such as the purchase of the land or the obtention of a patent over it. As
the land had become the private property of the locators, they had the right to transfer the same, as they
did, to Benguet and Atok. The Court of Appeals justified this by saying there is “no conflict of interest”
between the owners of the surface rights and the owners of the sub-surface rights. This is rather doctrine,
for it is a well-known principle that the owner of piece of land has rights not only to its surface but also to
everything underneath and the airspace above it up to a reasonable height. Under the aforesaid ruling,
the land is classified as mineral underneath and agricultural on the surface, subject to separate claims
of title. This is also difficult to understand, especially in its practical application.

NOTES:

The Court feels that the rights over the land are indivisible and that the land itself cannot be half
agricultural and half mineral. The classification must be categorical; the land must be either
completely mineral or completely agricultural. In the instant case, as already observed, the land
which was originally classified as forest land ceased to be so and became mineral — and completely
mineral — once the mining claims were perfected. As long as mining operations were being undertaken
thereon, or underneath, it did not cease to be so and become agricultural, even if only partly so, because
it was enclosed with a fence and was cultivated by those who were unlawfully occupying the surface.

The Regalian doctrine reserves to the State all minerals that may be found in public and even private
land devoted to "agricultural, industrial, commercial, residential or (for) any purpose other than mining."
Once minerals are discovered in the land, whatever the use to which it is being devoted at the time, such
use may be discontinued by the State to enable it to extract the minerals therein in the exercise of its
sovereign prerogative. The land is thus converted to mineral land and may not be used by any private

Page 15 of 35
PROPERTY | MBL | CASE DIGESTS 2020 – WEEK 2-1

party, including the registered owner thereof, for any other purpose that will impede the mining operations
to be undertaken therein. For the loss sustained by such owner, he is of course entitled to just
compensation under the Mining Laws or in appropriate expropriation proceedings

d. RECOVERY OF POSSESSION AND/OR OWNERSHIP


i. ACTIONS AVAILABLE TO OWNER
1. RECOVERY OF PERSONAL PROPERTY
a. REPLEVIN

i. DEFINITION

ARABESQUE INDUSTRIAL PHILS. VS. COURT OF APPEALS (G.R. NO. 101431 DECEMBER 14,
1992)

PETITIONER: RESPONDENT:

ARABESQUE INDUSTRIAL PHILS. (Lose) PNOC Dockyard and Engineering Corp. (PDEC)
Bought the tugboat from respondent, filed a writ of (Win)
replevin against the respondent

SPECIAL DETAILS: The name of the tugboat bought by the petitioner is MT Rover

LAW & PRINCIPLES:

A Writ of Replevin is a provisional remedy that provides instant relief to the person being deprived of
his property.

REPLEVIN; WRIT NOT ENFORCEABLE AGAINST A LAWFUL POSSESSOR. — Respondent Court of


Appeals correctly set aside the writ of replevin. Such writ cannot be properly directed against a lawful
possessor of a chattel, and the matter of ownership as well as incurring of additional lay day fees by the
continued detention of the boat by PDEC is therefore inconsequential.

FACTS:

Petitioner bought a tugboat belonging to respondent PDEC at a public auction who later notified that it
will impose lay day charges if the boat was not removed from its premises.

Petitioner did not remove it, instead it sought the services of PDEC to repair the boat. The amount to be
paid for the repairs by petitioner to PDEC rose to 1.68M pesos of which only 329k was paid. Petitioner
was then notified of the impending sale of the boat at a public auction.

Petitioner moved for the nullification of the public auction sale and prayed for a preliminary injunction
before the RTC which was granted.

Petitioner argues that he is not yet judicially declared insolvent and offered to pay 494k as a reasonable
and complete payment for its services.

PDEC opposed the motion. Petitioner, after posting a bong of 1M, filed a writ of replevin against PDEC.

Page 16 of 35
PROPERTY | MBL | CASE DIGESTS 2020 – WEEK 2-1

The trial court granted the writ and ordered the return of the boat to AIPI.

The Court of Appeals set aside the order and directed the return of the tugboat to PDEC on the ground
that the chattel was not wrongfully detained but possessed in the exercise of PDEC of a mechanic's lien
for its unpaid repair bills.

ISSUES: Whether the writ of replevin was properly issued by the court a quo, and whether the
interlocutory orders issued herein are appealable.

RULING: Petition denied.

The Supreme Court said:

Such writ cannot be properly directed against a lawful possessor of a chattel, and the matter of ownership as well
as incurring of additional lay day fees by the continued detention of the boat by PDEC is therefore inconsequential.
The requirement of posting a counterbond to reacquire possession of the chattel subject of the writ, does not apply
in the case at bar because that presupposes a previous valid writ. In the case before Us, however, the chattel was
ordered returned to PDEC because the writ was improperly issued. Definitely, it was not issued on the basis of the
non-posting of a counterbond.

As regards the second issue, interlocutory orders, because they do not dispose of the case on the merits, are not
appealable; consequently, they were correctly made subject of a petition for certiorari/prohibition before the Court of
Appeals under Rule 65 of the Rules of Court.

CHUA VS. COURT OF APPEALS (G.R. NO. 79021 MAY 17, 1993)

PETITIONER: RESPONDENT:

Romeo Chua (Lose) 2Lt. Dennis P. Canoy (Win)


Alleges ownership of the seized Isuzu dump truck seized ; Police officer who seized the Isuzu Dump
subject of the case of carnapping pending preliminary Truck by virtue of a search warrant and
investigation which was provisionally dismissed subsequent seizure

SPECIAL DETAILS:

LAW & PRINCIPLES:

REPLEVIN; DOES NOT LIE FOR PROPERTY IN CUSTODIA LEGIS. — It is a basic tenet of civil procedure that
replevin will not lie for property in custodia legis. A thing is in custodia legis when it is shown that it has been
and is subjected to the official custody of a judicial executive officer in pursuance of his execution of a legal writ
(Bagalihog vs. Fernandez, 198 SCRA 614 [1991]). The reason posited for this principle is that if it was otherwise,
there would be interference with the possession before the function of the law had been performed as to the process
under which the property was taken. Thus, a defendant in an execution or attachment cannot replevy goods in the
possession of an officer under a valid process, although after the, levy is discharged, an action to recover possession
will lie (Francisco, Revised Rules of Court in the Philippines: Provisional Remedies, p. 402 [1985]).|||

FACTS:

Page 17 of 35
PROPERTY | MBL | CASE DIGESTS 2020 – WEEK 2-1

1. Judge Lauro V. Francisco RTC Cebu branch 8, after examining 2Lt. Dennis P. Canoy and two
(2) other witnesses, issued a search warrant directing the immediate search of the premises of R.R.
Construction located at M.J. Cuenco Avenue, Cebu City, and the seizure of an Isuzu dump truck with
plate number GAP-175. At twelve noon of the same date, respondent Canoy seized the aforesaid vehicle
and took custody thereof.

2. RTC Cebu Branch 13: ACTION FOR Replevin/Sum of Money for the recovery of possession of
the same Isuzu dump truck was filed by petitioner against respondent Canoy and one "John Doe"

3. Petitioner Chua questioned the validity of the search warrant and the subsequent seizure of the
subject vehicle on the strength of the aforesaid search warrant. He alleges s lawful ownership and
possession of the subject vehicle; that he has not sold the subject vehicle to anyone; that he has not
stolen nor carnapped it, and that he has never been charged of the crime of carnapping or any other
crime for that matter.

4. Writ of REPLEVIN – ISSUED by RTC Ceby

5. Canoy filed a motion for the dismissal of the complaint and for the quashal of the writ of replevin
– DENIED. MR- DENIED: CA: Petition for Certiorari and Prohibition praying for the nullification of the
orders

6. Carnapping case was provisionally dismissed upon motion of Romeo Chua with the following
reservation: "without prejudice to its reopening once the issue of ownership is resolved"

7. CA reversed RTC decision - directed that possession of the subject vehicle be restored to Canoy.
Petitioner moved for a reconsideration of the decision, but the respondent court denied the same.

8. SC – certiorari.

ISSUES: Whether RTC erred when it ordered the transfer of possession of the property seized to
petitioner when the latter filed the action for replevin

RULING: Petition dismissed.

The Court citing Pagkalinawan vs. Gomez held;


It is a basic tenet of civil procedure that replevin will not lie for property in custodia legis. A thing is
in custodia legis when it is shown that it has been and is subjected to the official custody of a judicial
executive officer in pursuance of his execution of a legal writ (Bagalihog vs. Fernandez, 198 SCRA 614
[1991]). The reason posited for this principle is that if it was otherwise, there would be interference with
the possession before the function of the law had been performed as to the process under which the
property was taken. Thus, a defendant in an execution or attachment cannot replevy goods in the
possession of an officer under a valid process, although after the, levy is discharged, an action to recover
possession will lie (Francisco, Revised Rules of Court in the Philippines: Provisional Remedies, p. 402
[1985]).

NOTES:

2. RECOVERY OF REAL PROPERTY


a. FORCIBLE ENTRY AND UNLAWFUL DETAINER

i. NATURE OF THE ACTION/S

Page 18 of 35
PROPERTY | MBL | CASE DIGESTS 2020 – WEEK 2-1

ii. PRESCRIPTIVE PERIOD


iii. ISSUES INVOLVED
PERALTA-LABRADOR VS. BAGARIN (G.R. NO. 165177, AUGUST 25, 2005)

PETITIONER: RESPONDENT:

Lila V. Peralta Labrador (Lose) Silverio Bugarin, substituted by his widow,


Consolacion Bugarin (Win)
● plaintiff is the owner of a parcel of land
denominated as Cadastral lot No. 265|, ● Respondent contended that the area claimed
Zambales by petitioner is included in the 4,473 square
meter lot, covered by the Original Certificate of
● plaintiff has been in open, continuous, Title (OCT) No. P-13011; and that he has been
exclusive and adverse as well as notorious in continuous possession and occupation
possession of the said lot and in the concept thereof since 1955.
of an owner since she [acquired] it in 1976
until the time when defendant took possession
forcibly, two years ago|||

SUMMARY: Peralta-Labrador filed a case for recovery of possession and ownership with the MTC of
San Felipe Zambales against Bugarin, whom she says has been in unlawful possession of her lot. The
Court stated that since she filed the case two years after she learned of Bugarin’s possession of the lot,
it should have been filed with the RTC for a different cause of action. Her action for forcible entry had
prescribed after the 1-year mark, and is not eligible for a summary proceeding anymore.

LAW & PRINCIPLES: Cases for forcible entry should be filed with the MTC upon knowledge of information, if
past one year, then the case should be filed with the RTC for accion pauliana or accion reivindicatoria

FACTS:

● Petitioner alleged that she is the owner of Cadastral Lot No. 2650, 400 sqm, located at Barangay
Manglicmot, San Felipe, Zambales which she purchased from Sps. Pronto in 1976

In 1977, she was issued a tax declaration for the lot after she had paid the taxes due

1990: the DPWH constructed a road which traversed CLN. 2650 thereby separating 108sqm from
the rest of petitioner’s lot

1994: Respondent Bugarin forcibly took possession of the 108sqm and refused to vacate the same
despite the pleas of petitioner

January 18, 1996: Petitioner filed a case for “Recovery of Possession and Ownership” with the
MTC of San Felipe, Zambales

● MTC: Dismissed the case. Court was in favor of Respondent for failure of the plaintiff to establish the
preponderance of evidence of prior actual physical possession and present title over the lot in her favor
● RTC: Affirmed MTC decision
● CA: Modified RTC decision; deleted monetary awards

The motion for reconsideration filed by petitioner was denied. Hence the instant petition.

ISSUES:
1. Whether the CA erred in siding with Respondent

Page 19 of 35
PROPERTY | MBL | CASE DIGESTS 2020 – WEEK 2-1

2. Whether the MTC had jurisdiction over the case

RULING: Petition denied.


1. No.
Sec. 1, Rule 70 of the Revised Rules of Civil Procedure provides:

Section 1. Who may institute proceedings and when. – … a person deprived of the possession of any land
or building by force, intimidation, threat, strategy, or stealth… may at any time within one year after such
unlawful deprivation or withholding of possession, bring an action in the proper MTC against the
person/persons claiming under them, for the restitution of such possession, together with the damages and
costs.

2. No.

In Lopez v. David Jr., it was held that an action for forcible entry is a quieting process and the one year time
bar for filing a suit is in pursuance of the summary nature of the action

Thus, proceedings in the MTC shall be nullified if it improperly assumed jurisdiction of a case – in which a
case for unlawful deprivation or withholding of possession had exceeded one year

After the lapse of one year, the suit must be commenced in the RTC via an accion pauliana, a suit for
recovery of the right to possess or as an accion reivindicatoria, an action to recover ownership as well as
possession

The jurisdiction of a court is determined by the allegations of the complaint, thus in ascertaining whether or not the
action falls within the exclusive jurisdiction of the inferior courts, the averments of the complaint and the character of
the relief sought are to be examined.

In the case at bar, the plaintiff alleges that:

She has been in open, continuous, exclusive and adverse as well as notorious possession of the said lot
and in the concept of an owner since she acquired it in 1976, until the time when the defendant took
possession forcibly two years ago

It is clear in her averments that the unlawful possession occurred two years prior to her filing the complaint
for forcible entry in the MTC. The cause of action for forcible entry has prescribed, and the MTC had no
jurisdiction to entertain the case

The case should have been filed with the RTC, either for accion pauliana or accion reivindicatoria

PETITIONER: HUBERT NUÑEZ RESPONDENT: SLTEAS PHOENIX SOLUTIONS, INC.,


through its representative, CESAR SYLIANTENG
• that the subject parcel was left
idle and unguarded - claims of prior possession is clearly negated by the
fact that he had been in occupancy thereof since 1999.

SPECIAL DETAILS:

Subject matter: 635.50 square meter parcel of land situated at Calle Solana, Intramuros, Manila and
registered in the name of respondent SLTEAS Phoenix Solutions, Inc. under Transfer Certificate of Title
(TCT) No. 87556 of the Manila City Registry of Deeds

Page 20 of 35
PROPERTY | MBL | CASE DIGESTS 2020 – WEEK 2-1

LAW & PRINCIPLES:


Principle of physical possession

While prior physical possession is, admittedly, an indispensable requirement in forcible entry cases, the
dearth of merit in petitioner’s position is, however, evident from the principle that possession can be
acquired not only by material occupation, but also by the fact that a thing is subject to the action of one's
will or by the proper acts and legal formalities established for acquiring such right. Because possession
can also be acquired by juridical acts to which the law gives the force of acts of possession, e.g.,
donations, succession, execution and registration of public instruments, inscription of possessory
information titles and the like, it has been held that one need not have actual or physical occupation of
every square inch of the property at all times to be considered in possession.

FACTS:

• SLTEAS acquired the deed of assignment executed in its favor by the Spouses Ong Tiko and
Emerenciana Sylianteng, respondent was constrained to leave the subject parcel idle and
unguarded for some time due to important business concerns.
• In an ocular inspection conducted by respondent’s representatives revealed that the property
was already occupied by petitioner Nuñez and 21 other individuals with unjustified refusal to
heed its verbal demands to vacate the subject parcel
• SLTEAS filed a complaint for forcible entry with the METC.
• Contentions with the METC:
o Respondent:
▪ that thru its representatives and predecessors-in-interest, it had continuously
possessed the subject realty, over which it exercised all attributes of ownership,
including payment of real property taxes and other sundry expenses;
▪ that without the benefit of any lease agreement or possessory right, however,
petitioners and his co-defendants have succeeded in occupying the property by
means of strategy and stealth;
▪ that according to reliable sources, the latter had been in occupancy of the same
parcel since 1999.
o Petitioner:
▪ that the property occupied by him is owned by one Maria Ysabel Potenciano
Padilla Sylianteng, with whom he had concluded a subsisting lease agreement
over the same
▪ that in addition to respondent’s lack of cause of action against him, the MeTC
had no jurisdiction over the case for lack of prior demand to vacate and referral
of the controversy to the barangay authorities for a possible amicable
settlement.
• METC:
o FAVOR: SLTEAS
o Conclusion: the crowding of the residential units on the subject parcel rendered the
determination of its exact metes and bounds impossible. With the parties’ further failure
to abide by their agreement to cause a survey of the property thru an impartial surveyor
from the Office of the City Assessor or City Engineer, the record shows that respondent
submitted a survey plan prepared by Geodetic Engineer Joseph Padilla who determined
that petitioner was, indeed, occupying a portion of the subject parcel.
o Order: Vacate the premises, Pay 5,000 for each month from Oct 2003 until evacuation
of property, Attorney’s fees, and pay suit.
• RTC: Decision affirmed
• CA by way of a petition for review filed pursuant to Section 1, Rule 42 of the 1997 Rules of Civil
Procedure.

Page 21 of 35
PROPERTY | MBL | CASE DIGESTS 2020 – WEEK 2-1

• Conclusion: Although the dispossession took place more than one year from the illegal entry of
petitioner and his co-defendants, knowledge of the same was only acquired by petitioner in 2003
when the ocular inspection was made. The one-year prescriptive period should be reckoned
from the date of the actual entry on the land, the same however, does not hold true when entry
was made through stealth, in which case, the one year period is counted from the time the plaintiff
learned thereof.
o Neither may petitioner seek refuge in the alleged demand letter dated 31 july 1996 sent
by respondent’s counsel which sought his ouster from the subject premises. the
existence of this letter immaterial to the issue of illegal entry into the subject premises.
It cannot bind respondent who has no participation therein.
o Not once did petitioner refute the lack of knowledge on the part of respondent of the
alleged lease contract and their usurpation of the disputed property. Verily, granting that
a lease contract truly existed, respondent’s lack of knowledge of the lease contract and
the failure to register the same in the register of deeds cannot bind third parties like
respondent and therefore, withhold respondent’s right to institute the action for
ejectment.
o rtc committed no reversible error in admitting the evidence of respondent which consists
of the plan prepared by Geodetic Engineer Padilla.

ISSUES:

1. WON there was prior possession


2. WON there was no participation by respondent
3. WON the reliance with Engr Padilla is invalid
4. WON CA failed to apply Art. 1676 of the Civil Code in relation to the lease claims.

RULING:

1. (Read: principle section). The subject parcel was acquired by respondent by virtue of the 4 June
1999 Deed of Assignment executed in its favor by the Spouses Ong Tiko and Emerenciana
Sylianteng. Although it did not immediately put the same to active use, respondent appears to
have additionally caused the property to be registered in its name as of 27 February 2002 and
to have paid the real property taxes due thereon alongside the sundry expenses incidental
thereto. Viewed in the light of the foregoing juridical acts, it consequently did not matter that, by
the time respondent conducted its ocular inspection in October 2003, petitioner had already been
occupying the land since 1999. Ordinarily reckoned from the date of actual entry on the land, the
one year period is counted from the time the plaintiff acquired knowledge of the dispossession
when, as here, the same had been effected by means of stealth.
2. CA Decision affirmed on correctly brushing aside that that respondent had no participation in its
preparation, we find said demand letter of little or no use to petitioner’s cause in view of its non-
presentation before the metc.
3. The reliance with Engr. Padilla survey plan, as a rule, plaintiff has the burden of proving the
material allegations of the complaint which are denied by the defendant, and the defendant has
the burden of proving the material allegations in his case where he sets up a new matter. Given
the parties’ failure to make good on their agreement to cause a survey of the property thru an
impartial surveyor from the Office of the City Assessor or City Engineer, respondent’s submission
of said report was evidently for the purpose discharging the onus of proving petitioner’s
encroachment on the subject parcel, as alleged in the complaint. As the party asserting the
contrary proposition, petitioner cannot expediently disparage the admissibility and probative
value of said survey plan to compensate for his failure to prove his own assertions.

Page 22 of 35
PROPERTY | MBL | CASE DIGESTS 2020 – WEEK 2-1

4. In the absence of proof of his lessor’s title or respondent’s prior knowledge of said contract of
lease, petitioner’s harping over the same provision simply amounts to an implied admission that
the premises occupied by him lie within the metes and bounds of the subject parcel. Even then,
the resolution of said issue is clearly inappropriate since ejectment cases are summary actions
intended to provide an expeditious manner for protecting possession or right to possession
without involvement of title.

DELA CRUZ V. TAN TE

PETITIONER: Lourdes Dela Cruz was one of Reyes Fam RESPONDENT: Melba Tan Te, buyer
lessees, and she religiously paid rent over a portion of the lot for of the lot leased by Lourdes.
well over 40 years.

LAW & PRINCIPLES: (See notes)

FACTS:

• The Reyes familyowned the lot.


• A fire struck the premises and destroyed, among others, petitioner’s dwelling. After the fire,
petitioner and some tenants returned to the said lot and rebuilt their respective houses;
• Reyes family made several verbal demands on the remaining lessees, including petitioner, to
vacate the lot but the latter did not comply.
• Dela Cruz was served a written demand to vacate said lot but refused to leave. Despite, the
Reyes family did not initiate court proceedings against any of the lessees.
• Lot was then sold by the Reyeses to respondent Melba Tan Te. Respondent bought the lot in
question for residential purposes. Despite the sale, petitioner Dela Cruz did not give up the lot.
• Dela Cruz was sent a written demand to relinquish the premises which she ignored, prompting
respondent Tan Te to initiate conciliation proceedings at the barangay level.
• While respondent attempted to settle the dispute by offering financial assistance, petitioner
countered by asking PhP 500,000.00 for her house.
• Respondent rejected the counter offer which she considered unconscionable.
• Tan Te filed an ejectment complaint with damages before the Manila MeTC. The complaint
averred that: (1) the previous owners, the Reyeses were in possession and control of the
contested lot; (2) on November 26, 1996, the lot was sold to Tan Te; (3) prior to the sale, Dela
Cruz forcibly entered the property with strategy and/or stealth; (4) the petitioner unlawfully
deprived the respondent of physical possession of the property and continues to do so; and, (5)
the respondent sent several written demands to petitioner to vacate the premises but refused to
do so.
• Petitioner answer: (1) the MeTC had no jurisdiction over the case because it falls within the
jurisdiction of the RTC as more than one year had elapsed from petitioner’s forcible entry; (2)
she was a rent-paying tenant protected by PD 20; (3) her lease constituted a legal encumbrance
upon the property; and (4) the lot was subject of expropriation.
• METC: FAVOR: TAN TE
• RTC: Petition dismissed for having wrong jurisdiction.
• CA: FAVOR: TAN TE. Petition back to METC.

Page 23 of 35
PROPERTY | MBL | CASE DIGESTS 2020 – WEEK 2-1

ISSUES: WON METC HAS JURISDICTION

RULING:

The allegations in the complaint show that prior to the sale by Lino Reyes, representing the estate of his
wife Emerlinda Reyes, he was in possession and control of the subject lot but were deprived of said
possession when petitioner, by means of stealth and strategy, entered and occupied the same lot. These
circumstances imply that he had prior physical possession of the subject lot and can make up a forcible
entry complaint. On the other hand, the allegation that petitioner Dela Cruz was served several demands
to leave the premises but refused to do so would seem to indicate an action for unlawful detainer since
a written demand is not necessary in an action for forcible entry. It is a fact that the MeTC complaint was
filed on September 8, 1997 within one (1) year from the date of the last written demand upon petitioner
Dela Cruz on January 14, 1997. As previously discussed, the settled rule is jurisdiction is based on the
allegations in the initiatory pleading and the defenses in the answer are deemed irrelevant and immaterial
in its determination. However, we relax the rule and consider the complaint at bar as an exception in view
of the special and unique circumstances present. First, as in Ignacio v. CFI of Bulacan, the defense of
lack of jurisdiction was raised in the answer wherein there was an admission that petitioner Dela Cruz
was a lessee of the former owners of the lot, the Reyeses, prior to the sale to respondent Tan Te. The
fact that petitioner was a tenant of the predecessors-in-interest of respondent Tan Te is material to the
determination of jurisdiction. Since this is a judicial admission against the interest of petitioner, such
admission can be considered in determining jurisdiction. Second, the ejectment suit was filed with the
Manila MeTC on September 8, 1997 or more than nine (9) years ago. To dismiss the complaint would
be a serious blow to the effective dispensation of justice as the parties will start anew and incur additional
legal expenses after having litigated for a long time. Equitable justice dictates that allegations in the
answer should be considered to aid in arriving at the real nature of the action. Lastly, Section 6, Rule 1
of the Rules of Court clearly empowers the Court to construe Rule 70 and other pertinent procedural
issuances "in a liberal manner to promote just, speedy, and inexpensive disposition of every action and
proceeding."

Based on the complaint and the answer, it is apparent that the Tan Te ejectment complaint is after all a
complaint for unlawful detainer. It was admitted that petitioner Dela Cruz was a lessee of the Reyeses
for around four (4) decades. Thus, initially petitioner as lessee is the legal possessor of the subject lot by
virtue of a contract of lease. When fire destroyed her house, the Reyeses considered the lease
terminated; but petitioner Dela Cruz persisted in returning to the lot and occupied it by strategy and
stealth without the consent of the owners. The Reyeses however tolerated the continued occupancy of
the lot by petitioner. Thus, when the lot was sold to respondent Tan Te, the rights of the Reyeses, with
respect to the lot, were transferred to their subrogee, respondent Tan Te, who for a time
also tolerated the stay of petitioner until she decided to eject the latter by sending several demands, the
last being the January 14, 1997 letter of demand. Since the action was filed with the MeTC on September
8, 1997, the action was instituted well within the one (1) year period reckoned from January 14, 1997.
Hence, the nature of the complaint is one of unlawful detainer and the Manila MeTC had
jurisdiction over the complaint.

It is unequivocal that petitioner’s possession after she intruded into the lot after the fire—was by tolerance
or leniency of the Reyeses and hence, the action is properly an unlawful detainer case falling under the
jurisdiction of the Manila MeTC.

Notes to ruling:

Jurisdiction is the power or capacity given by the law to a court or tribunal to entertain, hear and determine
certain controversies. Jurisdiction over the subject matter is conferred by law.

Page 24 of 35
PROPERTY | MBL | CASE DIGESTS 2020 – WEEK 2-1

B. P. No. 129 Section 33 of Chapter III

Section 33 of Chapter III -- on Metropolitan Trial Courts, Municipal Trial Courts, and Municipal Circuit
Trial Courts of B. P. No. 129 provides: Section 33. Jurisdiction of Metropolitan Trial Courts, Municipal
Trial Courts and Municipal Circuit Trial Courts in civil cases.—Metropolitan Trial Courts, Municipal Trial
Courts, and Municipal Circuit Trial Courts shall exercise: x x x x (2) Exclusive original jurisdiction over
cases of forcible entry and unlawful detainer: Provided, That when, in such cases, the defendant raises
the question of ownership in his pleadings and the question of possession cannot be resolved without
deciding the issue of ownership, the issue of ownership shall be resolved only to determine the issue of
possession.

JURISDICTION OVER EJECTMENT PROCEEDINGS

Thus exclusive, original jurisdiction over ejectment proceedings (accion interdictal) is lodged with the first
level courts. This is clarified in Section 1, Rule 70 of the 1997 Rules of Civil Procedure that embraces an
action for forcible entry (detentacion), where one is deprived of physical possession of any land or
building by means of force, intimidation, threat, strategy, or stealth. In actions for forcible entry, three (3)
requisites have to be met for the municipal trial court to acquire jurisdiction. First, the plaintiffs must allege
their prior physical possession of the property. Second, they must also assert that they were deprived of
possession either by force, intimidation, threat, strategy, or stealth. Third, the action must be filed within
one (1) year from the time the owners or legal possessors learned of their deprivation of physical
possession of the land or building.

The other kind of ejectment proceeding is unlawful detainer (desahucio), where one unlawfully withholds
possession of the subject property after the expiration or termination of the right to possess. Here, the
issue of rightful possession is the one decisive; for in such action, the defendant is the party in actual
possession and the plaintiff’s cause of action is the termination of the defendant’s right to continue in
possession. The essential requisites of unlawful detainer are: (1) the fact of lease by virtue of a contract
express or implied; (2) the expiration or termination of the possessor’s right to hold possession; (3)
withholding by the lessee of the possession of the land or building after expiration or termination of the
right to possession; (4) letter of demand upon lessee to pay the rental or comply with the terms of the
lease and vacate the premises; and (5) the action must be filed within one (1) year from date of last
demand received by the defendant.

A person who wants to recover physical possession of his real property will prefer an ejectment suit
because it is governed by the Rule on Summary Procedure which allows immediate execution of the
judgment under Section 19, Rule 70 unless the defendant perfects an appeal in the RTC and complies
with the requirements to stay execution; all of which are nevertheless beneficial to the interests of the lot
owner or the holder of the right of possession.

Section 33 of Chapter III


On the other hand, Section 19, of Chapter II of B.P. No. 129 on Regional Trial Courts provides: Section
19. Jurisdiction in civil cases.—Regional Trial Courts shall exercise exclusive original jurisdiction: x x x x
(2) In all civil actions which involve the title to, or possession of, real property, or any interest therein,
except actions for forcible entry into and unlawful detainer of lands or buildings, original jurisdiction over
which is conferred upon Metropolitan Trial Courts, Municipal Trial Courts and Municipal Circuit Trial
Courts.

Two (2) kinds of action to recover possession of real property


Two (2) kinds of action to recover possession of real property which fall under the jurisdiction of the RTC
are: (1) the plenary action for the recovery of the real right of possession (accion publiciana) when the
dispossession has lasted for more than one year or when the action was filed more than one (1) year

Page 25 of 35
PROPERTY | MBL | CASE DIGESTS 2020 – WEEK 2-1

from date of the last demand received by the lessee or defendant; and (2) an action for the recovery of
ownership (accion reivindicatoria) which includes the recovery of possession.

These actions are governed by the regular rules of procedure and adjudication takes a longer period
than the summary ejectment suit.

Determine whether a complaint for recovery of possession falls under the jurisdiction of the metc
(first level court) or the RTC (second level court)

To determine whether a complaint for recovery of possession falls under the jurisdiction of the MeTC
(first level court) or the RTC (second level court), we are compelled to go over the allegations of the
complaint. The general rule is that what determines the nature of the action and the court that has
jurisdiction over the case are the allegations in the complaint. These cannot be made to depend upon
the defenses set up in the answer or pleadings filed by the defendant. This general rule however admits
exceptions. In Ignacio v. CFI of Bulacan, it was held "that while the allegations in the complaint make out
a case for forcible entry, where tenancy is averred by way of defense and is proved to be the real issue,
the case should be dismissed for lack of jurisdiction as the case should properly be filed with the then
Court of Agrarian Relations." The cause of action in a complaint is not what the designation of the
complaint states, but what the allegations in the body of the complaint define and describe. The
designation or caption is not controlling, more than the allegations in the complaint themselves are, for it
is not even an indispensable part of the complaint.

An ejectment complaint based on possession by tolerance of the owner, like the Tan Te
complaint, is a specie of unlawful detainer cases.

As early as 1913, case law introduced the concept of possession by tolerance in ejectment cases as
follows: It is true that the landlord might, upon the failure of the tenant to pay the stipulated rents, consider
the contract broken and demand immediate possession of the rented property, thus converting a legal
possession into illegal possession. Upon the other hand, however, the landlord might conclude to give
the tenant credit for the payment of the rents and allow him to continue indefinitely in the possession of
the property. In other words, the landlord might choose to give the tenant credit from month to month or
from year to year for the payment of their rent, relying upon his honesty of his financial ability to pay the
same. During such period the tenant would not be in illegal possession of the property and the landlord
could not maintain an action of desahucio until after he had taken steps to convert the legal possession
into illegal possession. A mere failure to pay the rent in accordance with the contract would justify the
landlord, after the legal notice, in bringing an action of desahucio. The landlord might, however, elect to
recognize the contract as still in force and sue for the sums due under it. It would seem to be clear that
the landlord might sue for the rents due and [unpaid, without electing to terminate the contract of tenancy;]
[w]hether he can declare the contract of tenancy broken and sue in an action desahucio for the
possession of the property and in a separate actions for the rents due and damages, etc.

CONCEPT OF POSSESSION BY TOLERANCE IN UNLAWFUL DETAINER


The concept of possession by tolerance in unlawful detainer cases was further refined and applied in
pertinent cases submitted for decision by 1966. The rule was articulated as follows: Where despite the
lessee’s failure to pay rent after the first demand, the lessor did not choose to bring an action in court but
suffered the lessee to continue occupying the land for nearly two years, after which the lessor made a
second demand, the one-year period for bringing the detainer case in the justice of the peace court
should be counted not from the day the lessee refused the first demand for payment of rent but from the
time the second demand for rents and surrender of possession was not complied with.

In Calubayan v. Pascual, a case usually cited in subsequent decisions on ejectment, the concept of
possession by tolerance was further elucidated as follows: In allowing several years to pass without

Page 26 of 35
PROPERTY | MBL | CASE DIGESTS 2020 – WEEK 2-1

requiring the occupant to vacate the premises nor filing an action to eject him, plaintiffs have
acquiesced to defendant’s possession and use of the premises. It has been held that a person
who occupies the land of another at the latter’s tolerance or permission, without any contract
between them, is necessarily bound by an implied promise that he will vacate upon demand,
failing which a summary action for ejectment is the proper remedy against them. The status of the
defendant is analogous to that of a lessee or tenant whose term of lease has expired but whose
occupancy continued by tolerance of the owner. In such a case, the unlawful deprivation or withholding
of possession is to be counted from the date of the demand to vacate.

SPOUSES MARIO OCAMPO and CARMELITA F. OCAMPO, vs. HEIRS OF BERNARDINO U.


DIONISIO, represented by ARTEMIO SJ. DIONISIO, G.R. No. 191101 October 1, 2014

PETITIONER: SPOUSES MARIO OCAMPO RESPONDENT: . HEIRS OF BERNARDINO U.


and CARMELITA F. OCAMPO DIONISIO, represented by ARTEMIO SJ. DIONISIO

LAW & PRINCIPLES:

• Section 18, Rule 70 of the Rules of Court expressly provides that a "judgment rendered
in an action for forcible entry or detainer shall be conclusive with respect to the
possession only and shall in no wise bind the title or affect the ownership of the land."

FACTS:
• Dionisio filed a complaint for forcible entry with the Municipal Trial Court (MTC) of Cardona,
Rizal, docketed as Civil Case No. 96-0031 (forcible entry case), against Mario Ocampo (Mario)
and Felix Ocampo (Felix).
• Dionisio sought to recover the possession of a portion of his property alleging that Mario and
Felix built a piggery thereon without his consent.
• Mario denied Dionisio’s allegation, claiming that the disputed parcel of land is owned by his wife,
Carmelita, who inherited the same from her father. Mario further claimed that they have been in
possession of the said parcel of land since 1969.
• MTC decision: Complaint dismiss.
o Dionisio failed to establish his prior possession of the disputed parcel of land.
• Dionisio died on September 27, 1997.Consequently, the heirs of Dionisio filed a complaint for
recovery of possession with the MTC. The respondents sought to recover the same portion of
the parcel of land.
o Respondent averred:
▪ that the subject property was acquired by Dionisio on February 10, 1945 when
he purchased the same from Isabelo Capistrano.
▪ Dionisio thereafter took possession of the subject property and was able to
obtain a free patent covering the subject property. OCT No. M-4559 was
subsequently issued in the name of Dionisio on December 22, 1987.
▪ that sometime in 1995, Mario constructed a piggery on a portion of the subject
property without their consent.
o petitioners maintained:
▪ that the subject parcel of land is owned by Carmelita, having acquired the same
through inheritance
▪ that they have been in possession thereof since 1969.

Page 27 of 35
PROPERTY | MBL | CASE DIGESTS 2020 – WEEK 2-1

▪ the petitioners claimed that the respondents’ complaint for recovery of


possession of the subject property is barred by res judicata in the light of the
finality of the decision in the forcible entry case.
• MTC: dismissed the complaint for being res judicata.
• RTC: MTC erred in dismissing the respondents’ complaint for recovery of possession oft he
subject property solely on the ground of res judicata.
o that the forcible entry case, only involves the question of who has a better right to the
possession of the subject property while the recovery of possession case not only
involves the right to the possession of the subject property, but the ownership thereof as
well.
o that a judgment rendered in a forcible entry case will not bar an action for recovery of
possession based on title or ownership since there is no identity of cause of action as
between the two cases.
o that the respondents were able to establish that the subject property is indeed part ofthe
parcel of land covered by OCT No. M-4559 registered in the name of Dionisio.
Considering that OCT No. M-4559 is registered under the name of Dionisio
o that the respondents, as sucessors-in-interestof Dionisio, are entitled to the possession
of the subject property as an attribute of their ownership over the same.
o that the petitioners failed to adduce sufficient evidence to support their claim that they
indeed own the subject property.
• CA: RTC decision affirmed. Respondents not barred by latches.

ISSUES: (1) whether the finality of the decision in the forcible entry case constitutes res judicata, which
would warrant the dismissal of the respondents’ complaint for recovery of possession; (2) whether the
respondents were able to establish their ownership of the subject property; and (3) whether the
respondents’ cause of action is already barred by laches.

RULING: Petition denied. Favor Dionisio.


1. RTC and CA is correct when it found that the forcible entry case only involves the issue of
possession over the subject property while the recovery of possession case puts in issue the
ownership of the subject property and the concomitant right to possess the same as an attribute
of ownership (See notes section for full discussion). The decision in the forcible entry case is
conclusive only as to the MTC’s determination that the petitioners are not liable for forcible entry
since the respondents failed to prove their prior physical possession; it is not conclusive as to
the ownership of the subject property. Besides, Section 18, Rule 70 of the Rules of Court
expressly provides that a "judgment rendered in an action for forcible entry or detainer shall be
conclusive with respect to the possession only and shall in no wise bind the title or affect the
ownership of the land."
2. The respondents were able to prove that they have a superior right over the subject property as
against the petitioners.1âwphi1 It is undisputed that the subject property is indeed covered by
OCT No. M-4559, which is registered in the name of Dionisio, the respondents’ predecessor-in-
interest. Between the petitioners’ unsubstantiated and self-serving claim that the subject property
was inherited by Carmelita from her father and OCT No. M-4559 registered in Dionisio’s name,
the latter must prevail. The respondents’ title over the subject property is evidence of their
ownership thereof. That a certificate of title serves as evidence of an indefeasible and
incontrovertible title to the property in favor of the person whose name appears therein and that
a person who has a Torrens title over a land is entitled to the possession thereof are fundamental
principles observed in this jurisdiction.
3. Equally untenable is the petitioners’ claim that the respondents’ right to recover the possession
of the subject property is already barred by laches. As owners of the subject property, the
respondents have the right to recover the possession thereof from any person illegally occupying
their property. This right is imprescriptible.

Page 28 of 35
PROPERTY | MBL | CASE DIGESTS 2020 – WEEK 2-1

NOTES:
• In an action for forcible entry and detainer, the only issue is possession in fact, or physical
possession of real property, independently of any claim of ownership that either party may put
forth in his pleading. If plaintiff can prove prior physical possession in himself, he may recover
such possession even from the owner, but, on the other hand, if he cannot prove such prior
physical possession, he has no right of action for forcible entry and detainer even if he should
be the owner of the property.
• Thus, even the MTC, in the forcible entry case, stressed that its determination is only limited to
the issue of who has "actual prior possession" of the subject property regardless of the ownership
of the same.
• On the other hand, the recovery of possession case is actually an accion reinvindicatoria or a
suit to recover possession of a parcel of land as an element of ownership. A perusal of the
complaint filed by the respondents in the recovery of possession case shows that the
respondents, as successors-in-interest of Dionisio, are asserting ownership of the subject
property and are seeking the recovery of possession thereof.
• A judgment rendered in a forcible entry case will not bar an action between the same parties
respecting title or ownership because between a case for forcible entry and an accion
reinvindicatoria, there is no identity of causes of action. Such determination does not bind the
title or affect the ownership of the land; neither is it conclusive of the facts therein found in a case
between the same parties upon a different cause of action involving possession.

CARMENCITA SUAREZ v. MR. AND MRS. FELIX E. EMBOY, JR. AND MARILOU P. EMBOY–
DELANTAR, G.R. No. 187944, March 12, 2014

PETITIONER: RESPONDENT:
• Carmencita’s complaint for unlawful detainer
• She purchased the subject lot from was fundamentally inadequate. There was
the Heirs of Vicente, who were then practically no specific averment as to when and
the registered owners. At the time of how possession by tolerance of the
the sale, respondents were occupying respondents began.
the subject lot. Thus, demanded that • Carmencita made a general claim that the
they vacate the property. respondents possessed “the property by mere
• Respondents’ refusal to comply with tolerance ‘with the understanding that they
the demand turned them into would voluntarily vacate the premises and
deforciants unlawfully withholding the remove their house(s) thereon upon demand by
possession of the subject lot from the owners’.” In Spouses Valdez, Jr. v. CA, the
Carmencita, the new owner, whose Court ruled that the failure of the complainants
recourse was to file a complaint for to allege key jurisdictional facts constitutive of
unlawful detainer. unlawful detainer is fatal and deprives the
MTCC of jurisdiction over the action.
• the issues of possession and ownership are
inseparably linked in the case at bar.
Carmencita’s complaint for ejectment was
based solely on her spurious title, which is
already the subject of the respondents’ petition
for nullification of partition of Lot No. 1907–A.

Page 29 of 35
PROPERTY | MBL | CASE DIGESTS 2020 – WEEK 2-1

SPECIAL DETAILS:
Anomalies discovered: forged signatures and alterations, in the execution of a series of deeds of partition
relative to Lot No. 1907–A.

FACTS:
Antecedents
• At the center of the dispute is a 222–square meter parcel of land, of the subdivision plan
situated in Barangay Duljo, Cebu City, and covered by Transfer Certificate of Title issued in the
name of Carmencita on February 9, 2005.
• The subject lot used to be a part of Lot No. 1907–A, which was partitioned in the following
manner among the heirs of Spouses Carlos Padilla (Carlos) and Asuncion Pacres (Asuncion). A
house occupied by respondents Felix and Marilou, stands in the subject lot and claim that their
mother had occupied the subject lot during her lifetime. They had thereafter stayed in the subject
lot for decades after inheriting the same from Claudia, who had in turn succeeded her own
parents, Carlos and Asuncion.
• Felix and Marilou were asked by their cousins, who are the Heirs of Vicente, to vacate the subject
lot and to transfer a landlocked portion sans a right of way. They refused to comply insisting that
Claudia’s inheritance pertained to Lot No. 1907–A–2.
• Not long after, they received from Carmencita’s counsel, Atty. Pareja, a demand letter, dated
February 23, 2004, requiring them to vacate the subject lot. They were informed that Carmencita
had already purchased on February 12, 2004 the subject lot from the former’s relatives.
• However, the respondents did not heed the demand. Instead, they examined the records
pertaining to the subject lot and uncovered possible anomalies.
• They filed before the RTC of Cebu City a complaint for nullification of the partition and for the
issuance of new TCTs covering the heirs’ respective portions of Lot No. 1907–A.
• Carmencita filed before the MTCC and against the respondents a complaint for unlawful
detainer, the origin of the instant petition.
o She alleged that she bought the subject lot from Remedios, Moreno, Veronica and
Dionesia, the registered owners thereof and the persons who allowed the respondents
to occupy the same by mere tolerance. As their successor–in–interest, she claimed her
entitlement to possession of the subject lot and the right to demand from the respondents
to vacate the same.
• MTC Decision: Carmencita’s favor.
• RTC Decision: RTC affirmed in its entirety the MTCC ruling.
• CA:
• Respondents arguments:
o That they have been occupying the subject lot in the concept of owners for several
decades. Carmencita, on the other hand, was a buyer in bad faith for having purchased
the property despite the notice of lis pendens clearly annotated on the subject lot’s title
o Invoked the doctrine enunciated in amagan v. Marayag that the pendency of another
action anchored on the issue of ownership justifies the suspension of an ejectment suit
involving the same real property. The foregoing is especially true in the case at bar
where the issue of possession is so interwoven with that of ownership. Besides, the
resolution of the question of ownership would necessarily result in the disposition of the
issue of possession.
o The deed of sale dated april 1, 2004, which was attached to the complaint for unlawful
detainer, bore tell–tale signs of being spurious.
▪ First, atty. Pareja’s demand letter sent to the respondents instead referred to a
deed of sale dated february 12, 2004.
▪ Secondly, teresita, who now lives in luzon and has been estranged from moreno
since the 1980s, was a signatory in the deed of sale.

Page 30 of 35
PROPERTY | MBL | CASE DIGESTS 2020 – WEEK 2-1

▪ Thirdly, a certain veronida padilla, a fictitious person, also signed the deed of
sale as among the vendors, but she, too, was impleaded as a co–defendant in
the ejectment suit.
▪ Fourthly, the deed was only registered the following year after its supposed
execution.
o The heirs of vicente, who had allegedly sold the subject lot to carmencita, had never
physically occupied the same. Hence, there was no basis at all for carmencita’s claim
that the respondents’ possession of the subject lot was by mere tolerance of the alleged
owners.
o Carmencita’s complaint lacked a cause of action.

CA: DECISIONS REVERSED.

• It reveals that the action was neither one of forcible entry nor unlawful detainer but essentially
involved an issue of ownership which must be resolved in an accion reivindicatoria.
o It did not characterize alleged entry into the land: whether the same was legal or
illegal. It did not state how respondents entered the land and constructed a house
thereon. It was also silent on whether respondent’s possession became legal before
she demanded from them to vacate the land.
o The complaint merely averred that their relatives previously owned the lot and that after
purchased, she, as its new owner, demanded to vacate the land. Moreover, it is
undisputed that respondents and their ancestors have been occupying the land for
several decades already.
o no averment as to how or when [Carmencita’s] predecessors tolerated [the
respondents’] possession of the land. Consequently, there was no contract to speak of,
whether express or implied, between [the respondents], on one hand, and [Carmencita]
or her predecessors, on the other, as would qualify [the respondents’] possession of the
land as a case of unlawful detainer.
o Neither was it alleged that [the respondents] took possession of the land through force,
intimidation, threat, strategy or stealth to make out a case of forcible entry.
• In any event, [Carmencita] cannot legally assert that [the respondents’] possession of the land
was by mere tolerance. This is because [Carmencita’s] predecessors–in–interest did not yet
own the property when [Claudia] took possession thereof. Take note that [Carmencita’s]
predecessors–in–interest merely stepped into the shoes of their parents who were also co–heirs
of [Claudia]. Finally, to categorize a cause of action as one constitutive of unlawful detainer,
plaintiff’s supposed acts of tolerance must have been present from the start of the possession
which he later seek[s] to recover. This is clearly wanting in the case at bar.
• Indeed, when the complaint fails to aver facts constitutive of forcible entry or unlawful detainer,
as where it does not state how entry was effected or how and when dispossession started, as in
the case at bar, the remedy should either be an accion publiciana or an accion reivindicatoria in
the proper RTC. If [Carmencita] is truly the owner of the subject property and she was unlawfully
deprived of the real right of possession or ownership thereof, she should present her claim before
the RTC in an accion publiciana or an accion reivindicatoria, and not before the municipal trial
court in a summary proceeding of unlawful detainer or forcible entry.

ISSUES:
1. Whether or not Carmencita’s complaint against the respondents had sufficiently alleged and
proven a cause of action for unlawful detainer.
2. Whether or not the pendency of the respondents’ petition for nullification of partition of Lot No.
1907–A and for the issuance of new certificates of title can abate Carmencita’s ejectment suit.

Page 31 of 35
PROPERTY | MBL | CASE DIGESTS 2020 – WEEK 2-1

RULING: Petition DENIED. No error in CA in dismissing Carmencita’s complaint for unlawful detainer.
(To understand the ruling, it is advisable to read the notes section first).

1. Carmencita had not amply alleged and proven that all the requisites for unlawful detainer
are present in the case at bar. The first requisite mentioned above is markedly
absent. Carmencita failed to clearly allege and prove how and when the respondents entered
the subject lot and constructed a house upon it. Carmencita was likewise conspicuously silent
about the details on who specifically permitted the respondents to occupy the lot, and how and
when such tolerance came about. Instead, Carmencita cavalierly formulated a legal
conclusion, sans factual substantiation, that (a) the respondents’ initial occupation of the subject
lot was lawful by virtue of tolerance by the registered owners, and (b) the respondents became
deforciants unlawfully withholding the subject lot’s possession after Carmencita, as purchaser
and new registered owner, had demanded for the former to vacate the property. It is worth noting
that the absence of the first requisite assumes even more importance in the light of the
respondents’ claim that for decades, they have been occupying the subject lot as owners thereof.
2. As an exception to the general rule, the respondents’ petition for nullification of the
partition of Lot No. 1907–A can abate Carmencita’s suit for unlawful detainer. The doctrines
enunciated in Amagan squarely applicable to the instant petition for reasons discussed
hereunder. Further, should Carmencita’s complaint be granted, the respondents’ house, which
has been standing in the subject lot for decades, would be subject to demolition. The foregoing
circumstances, thus, justify the exclusion of the instant petition from the purview of the general
rule.

NOTES:
CA Discussion:
• Section 1, Rule 70 of the Rules of Court provides:
Section 1. Who may institute proceedings, and when.—Subject to the provisions of the next
succeeding section, a person deprived of the possession of any land or building by force,
intimidation, threat, strategy, or stealth, or a lessor, vendor, vendee, or other person against
whom the possession of any land or building is unlawfully withheld after the expiration or
termination of the right to hold possession, by virtue of any contract, express or implied, or the
legal representatives or assigns of any such lessor, vendor, vendee, or other person, may, at
any time within one (1) year after such unlawful deprivation or withholding of possession, bring
an action in the proper Municipal Trial Court against the person or persons unlawfully withholding
or depriving of possession, or any person or persons claiming under them, for the
restitution of such possession, together with damages and costs.
• The distinction between forcible entry and unlawful detainer was lucidly explained
in Sarmiento vs. Court of Appeals,: Forcible entry and unlawful detainer cases are two distinct
actions defined in Section 1, Rule 70 of the Rules of Court. [In] forcible entry, one is deprived of
physical possession of land or building by means of force, intimidation, threat, strategy, or
stealth. In unlawful detainer, one unlawfully withholds possession thereof after the expiration or
termination of his right to hold possession under any contract, express or implied. In forcible
entry, the possession is illegal from the beginning and the basic inquiry centers on who has the
prior possession de facto. In unlawful detainer, the possession was originally lawful but became
unlawful by the expiration or termination of the right to possess, hence the issue of rightful
possession is decisive for, in such action, the defendant is in actual possession and the plaintiffs
cause of action is the termination of the defendant’s right to continue in possession.
• What determines the cause of action is the nature of defendant’s entry into the land. If
the entry is illegal, then the action which may be filed against the intruder within one (1) year
therefrom is forcible entry. If, on the other hand, the entry is legal but the possession thereafter
became illegal, the case is one of unlawful detainer which must be filed within one (1) year from
the date of the last demand.
• Munoz vs. Court of Appeals enunciated: For even if he is the owner, possession of the property
cannot be wrested from another who had been in possession thereof for more than twelve (12)

Page 32 of 35
PROPERTY | MBL | CASE DIGESTS 2020 – WEEK 2-1

years through a summary action for ejectment. Although admittedly[,] petitioner may validly
claim ownership based on the muniments of title it presented, such evidence does not
responsibly address the issue of prior actual possession raised in a forcible entry case. It must
be stated that regardless of actual condition of the title to the property, the party in peaceable
quiet possession shall not be turned out by a strong hand, violence or terror. Thus, a party who
can prove prior possession can recover such possession even against the owner himself.
Whatever may be the character of his prior possession, if he has in his favor priority in time, he
has the security that entitles him to remain on the property until he is lawfully ejected by a person
having a better right by accion publiciana or accion reivindicatoria.

Discussion to first issue:


• “Without a doubt, the registered owner of real property is entitled to its possession. However,
the owner cannot simply wrest possession thereof from whoever is in actual occupation of the
property. To recover possession, he must resort to the proper judicial remedy and, once he
chooses what action to file, he is required to satisfy the conditions necessary for such action to
prosper.”
• In Spouses Valdez, Jr., the Court is instructive anent the three kinds of actions available to
recover possession of real property, viz: (a) accion interdictal; (b) accion publiciana; and
(c) accion reivindicatoria.
o Accion interdictal comprises two distinct causes of action, namely, forcible entry
(detentacion) and unlawful detainer (desahuico) [sic]. In forcible entry, one is deprived
of physical possession of real property by means of force, intimidation, strategy, threats,
or stealth whereas in unlawful detainer, one illegally withholds possession after the
expiration or termination of his right to hold possession under any contract, express or
implied. The two are distinguished from each other in that in forcible entry, the
possession of the defendant is illegal from the beginning, and that the issue is which
party has prior de facto possession while in unlawful detainer, possession of the
defendant is originally legal but became illegal due to the expiration or termination of the
right to possess. The jurisdiction of these two actions, which are summary in nature, lies
in the proper municipal trial court or metropolitan trial court. Both actions must be
brought within one year from the date of actual entry on the land, in case of forcible
entry, and from the date of last demand, in case of unlawful detainer. The issue in said
cases is the right to physical possession.
o Accion publiciana is the plenary action to recover the right of possession which should
be brought in the proper regional trial court when dispossession has lasted for more than
one year. It is an ordinary civil proceeding to determine the better right of possession of
realty independently of title. In other words, if at the time of the filing of the complaint
more than one year had elapsed since defendant had turned plaintiff out of possession
or defendant’s possession had become illegal, the action will be, not one of the forcible
entry or illegal detainer, but an accion publiciana. On the other hand, accion
reivindicatoria is an action to recover ownership also brought in the proper regional trial
court in an ordinary civil proceeding.
• In a complaint for unlawful detainer, the following key jurisdictional facts must be alleged and
sufficiently established: (1) initially, possession of property by the defendant was by contract with
or by tolerance of the plaintiff; (2) eventually, such possession became illegal upon notice by
plaintiff to defendant of the termination of the latter’s right of possession; (3) thereafter, the
defendant remained in possession of the property and deprived the plaintiff of the enjoyment
thereof; and (4) within one year from the last demand on defendant to vacate the property, the
plaintiff instituted the complaint for ejectment.
• Again, this Court stresses that to give the court jurisdiction to effect the ejectment of an occupant
or deforciant on the land, it is necessary that the complaint must sufficiently show such a
statement of facts as to bring the party clearly within the class of cases for which the statutes
provide a remedy, without resort to parol testimony, as these proceedings are summary in

Page 33 of 35
PROPERTY | MBL | CASE DIGESTS 2020 – WEEK 2-1

nature. In short, the jurisdictional facts must appear on the face of the complaint. When the
complaint fails to aver facts constitutive of forcible entry or unlawful detainer, as where it does
not state how entry was effected or how and when dispossession started, the remedy should
either be an accion publiciana or accion reivindicatoria.

Discussion to second issue:

• In Amagan, the Court is emphatic that: As a general rule, therefore, a pending civil action
involving ownership of the same property does not justify the suspension of ejectment
proceedings. “The underlying reasons for the above ruling were that the actions in the Regional
Trial Court did not involve physical or de facto possession, and, on not a few occasions, that the
case in the Regional Trial Court was merely a ploy to delay disposition of the ejectment
proceeding, or that the issues presented in the former could quite as easily be set up as defenses
in the ejectment action and there resolved.”
• Only in rare instances is suspension allowed to await the outcome of the pending civil
action. One such exception is Vda. de Legaspi v. Avendaño, wherein the Court declared: “x x
x. Where the action, therefore, is one of illegal detainer, as distinguished from one of forcible
entry, and the right of the plaintiff to recover the premises is seriously placed in issue in a proper
judicial proceeding, it is more equitable and just and less productive of confusion and disturbance
of physical possession, with all its concomitant inconvenience and expenses. For the Court in
which the issue of legal possession, whether involving ownership or not, is brought to restrain,
should a petition for preliminary injunction be filed with it, the effects of any order or decision in
the unlawful detainer case in order to await the final judgment in the more substantive case
involving legal possession or ownership. It is only where there has been forcible entry that as a
matter of public policy the right to physical possession should be immediately set at rest in favor
of the prior possession regardless of the fact that the other party might ultimately be found to
have superior claim to the premises involved, thereby to discourage any attempt to recover
possession thru force, strategy or stealth and without resorting to the courts.” x x x Indisputably,
the execution of the MCTC Decision would have resulted in the demolition of the house subject
of the ejectment suit; thus, by parity of reasoning, considerations of equity require the suspension
of the ejectment proceedings.
o We note that, like Vda. de Legaspi, the respondent’s suit is one of unlawful detainer and
not of forcible entry. And most certainly, the ejectment of petitioners would mean a
demolition of their house, a matter that is likely to create the “confusion, disturbance,
inconveniences and expenses” mentioned in the said exceptional case.
o We should stress that respondent’s claim to physical possession is based not on an
expired or a violated contract of lease, but allegedly on “mere tolerance.” Without in any
way prejudging the proceedings for the quieting of title, we deem it judicious under the
present exceptional circumstances to suspend the ejectment case.
• The Court then quoted with favor the following portion of the Decision dated July 8, 1997, penned
by Associate Justice Artemio G. Tuquero in CA–G.R. No. 43611–SP, from which
the Amagan case sprang:
o “ONE. Private respondent Teodorico T. Marayag anchors his action for unlawful detainer
on the theory that petitioners’ possession of the property in question was by mere
tolerance. However, in answer to his demand letter dated April 13, 1996 x x x, petitioners
categorically denied having any agreement with him, verbal or written, asserting that
they are ‘owners of the premises we are occupying at 108 J.P. Rizal Street, San Vicente,
Silang, Cavite.’ In other words, it is not merely physical possession but ownership as
well that is involved in this case.[”]
o “TWO. In fact, to protect their rights to the premises in question, petitioners filed an action
for reconveyance, quieting of title and damages against private respondents, docketed
as Civil Case No. TG–1682 of the Regional Trial Court, Branch 18, Tagaytay City. The
issue of ownership is squarely raised in this action. Undoubtedly, the resolution of this

Page 34 of 35
PROPERTY | MBL | CASE DIGESTS 2020 – WEEK 2-1

issue will be determinative of who is entitled to the possession of the premises in


question.[”]
o “THREE. The immediate execution of the judgment in the unlawful detainer case will
include the removal of the petitioners’ house [from] the lot in question.[”]
o “To the mind of the Court it is injudicious, nay inequitable, to allow demolition of
petitioners’ house prior to the determination of the question of ownership [of] the lot on
which it stands.”

Page 35 of 35

You might also like